In order to reduce traffic congestion and raise revenue for the city, the mayor plans to implement a charge of $10 pe...

Julie-V on September 11, 2019

Answer Explanation

Hi LSAT Max, I was able to choose (A) but wasn't 100% confident about it. Could you help explain this question? Many thanks in advance!

Reply
Create a free account to read and take part in forum discussions.

Already have an account? log in

Irina on September 11, 2019

@Julie,

The argument tells us that to reduce traffic congestion, the mayor plans to implement $10 charge to drive to downtown. Payment of the charge will be enforced using a highly sophisticated system, but it will not be ready until the end of next year. Without the system, no one is going to pay the charge. Therefore, when the mayor's plan is first implemented, payment of the charge will not be effectively enforced.

To argument provides a set of premises:

payment is enforced using a sophisticated system
system is not ready till the end of next year
no system -> mass evasion of the charge
plan implemented

and concludes that:

payment is not effectively enforced.

The question asks us to identify a required assumption. There are a number of assumptions we can think of before even looking at the answer choices.

(1) Effective enforcement requires that the camera system is in place
No camera system in place means the enforcement is ineffective.

(2) Plan is first implemented before the end of next year meaning no camera system is yet in place.

The correct answer choice (A) is a perfect match for our second assumption. If the plan is implemented before the end of next year, it means the enforcement system is not yet ready, and the enforcement is, therefore, ineffective.

implemented before the end of next year -> no camera system -> no effective enforcement

Let me know if this makes sense and if you have any further questions.